subject
Business, 11.02.2020 20:02 greenclonetroop6q2tg

Suppose your firm has a marginal revenue given by the equation MR equals 10 minus Upper Q, where Q is the quantity produced and sold. This means that the seventh unit of output brings in 10 minus 7 equals $ 3 of additional revenue. The marginal cost for your firm is given by the equation MC equals 2 plus Upper Q. This means that the seventh unit of output increases the marginal cost by 2 plus 7 equals $ 9.

ansver
Answers: 1

Another question on Business

question
Business, 21.06.2019 20:20
Atoy manufacturer makes its own wind-up motors, which are then put into its toys. while the toy manufacturing process is continuous, the motors are intermittent flow. data on the manufacture of the motors appears below.annual demand (d) = 50,000 units daily subassembly production rate = 1,000setup cost (s) = $85 per batch daily subassembly usage rate = 200carrying cost = $.20 per unit per year(a) to minimize cost, how large should each batch of subassemblies be? (b) approximately how many days are required to produce a batch? (c) how long is a complete cycle? (d) what is the average inventory for this problem? (e) what is the total annual inventory cost (holding plus setup) of the optimal behavior in this problem?
Answers: 2
question
Business, 22.06.2019 01:40
Suppose general motors demands labor according to the labor demand function 푤푤= 40−0. 5퐸퐸, where 푤푤 is the hourly wage and 퐸퐸 is the number of employees. the united auto workers union has a utility function given by 푈푈=푊푊∗퐸퐸. a.in 1984, the united auto workers union started negotiations with general motors by assuming that they were a monopoly union. find the wage and employment demands that the united auto workers union would have demanded before any bargaining began. b.if general motors and the united auto workers union both had excellent bargaining representatives, would this be the final labor contract? if not, then explain in words and graphically where they would end up after the bargaining process.
Answers: 1
question
Business, 22.06.2019 15:20
Garfield corporation is considering building a new plant in canada. it predicts sales at the new plant to be 50,000 units at $5.00/unit. below is a listing of estimated expenses. category total annual expenses % of annual expense that are fixed materials $50,000 10% labor $90,000 20% overhead $40,000 30% marketing/admin $20,000 50% a canadian firm was contracted to sell the product and will receive a commission of 10% of the sales price. no u.s. home office expenses will be allocated to the new facility. the contribution margin ratio for garfield corporation is
Answers: 2
question
Business, 22.06.2019 18:30
Order these statements in the correct order to fill in the central idea and key points for a chronological speech. question 22 options: there are several steps that someone must take to become a doctor. finally, you will need to get a medical license, and become board certified in your specialty area. then, you must get admitted into to medical school and earn a medical degree. next, you will need to complete a residency. first you must earn a bachelors degree.
Answers: 2
You know the right answer?
Suppose your firm has a marginal revenue given by the equation MR equals 10 minus Upper Q, where Q i...
Questions
question
Advanced Placement (AP), 03.02.2021 22:20
question
Mathematics, 03.02.2021 22:20
question
Mathematics, 03.02.2021 22:20
question
Computers and Technology, 03.02.2021 22:20
Questions on the website: 13722363